Proofing question (1 Viewer)

CM_Tutor

Moderator
Moderator
Joined
Mar 11, 2004
Messages
2,644
Gender
Male
HSC
N/A
The case of the AM-GM-HM inequality states that, so long as



It can be proven easily and every MX2 student should be able to do this from memory:

Let and be two positive real numbers. Since they must then have positive square roots and all non-zero reals have a positive square, it follows that:

AM-GM:



GM-HM:



Combining these gives the full AM-GM-HM inequality for .
 

CM_Tutor

Moderator
Moderator
Joined
Mar 11, 2004
Messages
2,644
Gender
Male
HSC
N/A
Question 25(i)

If , and , show that

Solution using AM-HM:

Let and in the statements in the previous post:



Solution using only AM-GM:

Let and in the statements in the previous post:



Now, Let and in the statements in the previous post (as we can substitute any values that are positive):

 

AbstractBlade

New Member
Joined
Sep 8, 2020
Messages
21
Gender
Male
HSC
2021
Question 25(i)

If , and , show that

Solution using AM-HM:

Let and in the statements in the previous post:



Solution using only AM-GM:

Let and in the statements in the previous post:



Now, Let and in the statements in the previous post (as we can substitute any values that are positive):

does this work for part (ii). I tried it and got something else
 

CM_Tutor

Moderator
Moderator
Joined
Mar 11, 2004
Messages
2,644
Gender
Male
HSC
N/A
Question 25(ii)

If , and , show that

Solution using only AM-GM:

Let and in the statements in the previous post:



Now, Let and in the statements in the previous post (as we can substitute any values that are positive):



Note: I thought than an AM-HM proof would be easier but it turned messy...

Solution using AM-HM:

Let and in the statements in the previous post:



Now, we need to simplify:



and then recognise that the , which is easily shown using the AMGM inequality:



So, returning to (*), we have:

 

AbstractBlade

New Member
Joined
Sep 8, 2020
Messages
21
Gender
Male
HSC
2021
Question 25(ii)

If , and , show that

Solution using only AM-GM:

Let and in the statements in the previous post:



Now, Let and in the statements in the previous post (as we can substitute any values that are positive):



Note: I thought than an AM-HM proof would be easier but it turned messy...

Solution using AM-HM:

Let and in the statements in the previous post:



Now, we need to simplify:



and then recognise that the , which is easily shown using the AMGM inequality:



So, returning to (*), we have:

thank you
 

Users Who Are Viewing This Thread (Users: 0, Guests: 1)

Top